how many 6-digit numbers can be created using8, 0, 1, 3, 7, and 5 if each number is used only once?

Answers

Answer 1

Answer:

600 numbers

Step-by-step explanation:

For six-digit numbers, we need to use all digits 8,0,1,3,7,5 each once.

However, 0 cannot be used as the first digit, because it would make a 5-digit number.

Therefore

there are 5 choices for the first digit (exclude 0)

there are 5 choices for the first digit (include 0)

there are 4 choices for the first digit

there are 3 choices for the first digit

there are 2 choices for the first digit

there are 1 choices for the first digit

for a total of 5*5*4*3*2*1 = 600 numbers


Related Questions

evaluate 15.2% of a 726 + 12.8% of 673

Answers

Answer:

196.496

Step-by-step explanation:

0.152x726+0.128x673

110.352+86.144

=196.496

Arnold made the following grades on his quizzes and assignments: 80, 92, 88, 90, 75, 38, 92, 95 2. Arnold wants to present his scores as advantageously as possible. Should he use the mean or the median of this data set? What other strategies could he employ to yield a more favorable measure of center?

Answers

Answer:

Step-by-step explanation:

If he were to find the mean then he would need to add all the numbers up and divide by the number of numbers. The answer of this would 69.1

If you find the median you would need to put the numbers in order,

2, 38, 75, 80, 88, 90, 92, 92, 95, then find the middle which would be 88.

So the better option would be finding the median. I think that this would be the best way to get the the most favorable measure of center.

I hope this helps.

Coordinate plane with two lines graphed. The equations of the lines are y equals negative two-thirds x plus four and the other line is y equals two-thirds x. Determine the number of solutions the system of linear equations has and the solution(s) to the equations represented by these two lines? The system of equations has 0 solutions, because the graph has no point of intersection. The system of equations has infinite number of solutions and all real numbers satisfy both equations. The system of equations has 1 solution and it is (3, 2). The system of equations has 1 solution and it is (3, 0).

Answers

Answer:

Step-by-step explanation:

y = -2/3x + 4

y = 2/3x

2/3x = -2/3x + 4

4/3x = 4

4x = 12

x = 3

y = 2/3(3)

y = 2

(3,2) one solution

option 3

The domain of this function is {-12, -6, 3, 15}. y=-2/3x+7 Complete the table based on the given domain.

Answers

Answer:

Step-by-step explanation:

Domain of a function represents the set of x-values (input values) and y-values (output values) of the function represent the Range of the function.

Given function is,

[tex]y=-\frac{2}{3}x+7[/tex]

If Domain (input values) of this function is,

{-12, -6, 3, 15}

Table for the input-output values of this function,

x        -6         3        15       -12

y        11         5         -3        15

Answer:

Step-by-step explanation:

The pregnancy length in days for a population of new mothers can be approximated by a normal distribution with a mean of days and a standard deviation of days. ​(a) What is the minimum pregnancy length that can be in the top ​% of pregnancy​ lengths? ​(b) What is the maximum pregnancy length that can be in the bottom ​% of pregnancy​ lengths? ​(a) The minimum pregnancy length is 280 days.

Answers

Answer:

(a) 283 days

(b) 248 days

Step-by-step explanation:

The complete question is:

The pregnancy length in days for a population of new mothers can be approximated by a normal distribution with a mean of 268 days and a standard deviation of 12 days. ​(a) What is the minimum pregnancy length that can be in the top 11​% of pregnancy​ lengths? ​(b) What is the maximum pregnancy length that can be in the bottom ​5% of pregnancy​ lengths?

Solution:

The random variable X can be defined as the pregnancy length in days.

Then, from the provided information [tex]X\sim N(\mu=268, \sigma^{2}=12^{2})[/tex].

(a)

The minimum pregnancy length that can be in the top 11​% of pregnancy​ lengths implies that:

P (X > x) = 0.11

⇒ P (Z > z) = 0.11

z = 1.23

Compute the value of x as follows:

[tex]z=\frac{x-\mu}{\sigma}\\\\1.23=\frac{x-268}{12}\\\\x=268+(12\times 1.23)\\\\x=282.76\\\\x\approx 283[/tex]

Thus, the minimum pregnancy length that can be in the top 11​% of pregnancy​ lengths is 283 days.

(b)

The maximum pregnancy length that can be in the bottom ​5% of pregnancy​ lengths implies that:

P (X < x) = 0.05

⇒ P (Z < z) = 0.05

z = -1.645

Compute the value of x as follows:

[tex]z=\frac{x-\mu}{\sigma}\\\\-1.645=\frac{x-268}{12}\\\\x=268-(12\times 1.645)\\\\x=248.26\\\\x\approx 248[/tex]

Thus, the maximum pregnancy length that can be in the bottom ​5% of pregnancy​ lengths is 248 days.

Jorge’s monthly bill from his Internet service provider was $25. The service provider charges a base rate of $15 per month plus $1 for each hour that the service is used. Find the number of hours that Jorge was charged for that month.

Answers

Answer:

10 hours for the month

Step-by-step explanation:

What you know: The total amount Jorge was charged for the month was $25

The base rate is $15

He gets charged $1 per each hour

Setting it up:

15+1h=25

(the 15 is the base rate, plus the 1 dollar per hour (h) which both add to the total of 25 dollars for the month)

Subtract 15 from both sides of the equation to get your variable by itself

1h=10

then divide the 1 on both sides to get h (hours) by itself

h=10

And there's your answer, 10 is the number of hours that Jorge was charged for the month

Hopefully this helped :))

Answer:

10

Step-by-step explanation:

$25 - $15 = $10

and its $1 per hour so the answer is 10hrs

9 ÷ (-4 1/2) = L.
What is L.​

Answers

Answer:

[tex]\huge\boxed{L = -2}[/tex]

Step-by-step explanation:

L = 9 ÷ [tex](-4 \frac{1}{2} )[/tex]

L = 9 ÷ [tex](-\frac{9}{2} )[/tex]

L = 9 × [tex](-\frac{2}{9} )[/tex]

L = 1× (-2)

L = -2

●✴︎✴︎✴︎✴︎✴︎✴︎✴︎✴︎❀✴︎✴︎✴︎✴︎✴︎✴︎✴︎✴︎✴︎●

Hi my lil bunny!

❧⎯⎯⎯⎯⎯⎯⎯⎯⎯⎯⎯⎯⎯⎯⎯⎯⎯⎯⎯⎯⎯⎯⎯⎯⎯⎯⎯⎯⎯⎯⎯⎯⎯⎯⎯⎯⎯⎯☙

[tex]\boxed {l = \frac{-18}{41} }[/tex]

[tex]\frac{\frac{9}{-41} }{2} = l[/tex]

Simplifies to:

[tex]\frac{-18}{41} = l[/tex]

Let's solve your equation step-by-step.

[tex]\frac{-18}{41} = l[/tex]

Step 1: Flip the equation.

[tex]l = \frac{-18}{41}[/tex]

So your answer would be : [tex]\boxed {l = \frac{-18}{41} }[/tex]

❧⎯⎯⎯⎯⎯⎯⎯⎯⎯⎯⎯⎯⎯⎯⎯⎯⎯⎯⎯⎯⎯⎯⎯⎯⎯⎯⎯⎯⎯⎯⎯⎯⎯⎯⎯⎯⎯⎯☙

●✴︎✴︎✴︎✴︎✴︎✴︎✴︎✴︎❀✴︎✴︎✴︎✴︎✴︎✴︎✴︎✴︎✴︎●

Have a great day/night!

❀*May*❀

I NEED HELP ANSWERING THESE QUESTIONS FIRST ANSWER GET BRAINLIEST!

Answers

Answer:

3 - b=12

4- b=14.1

Step-by-step explanation:

Area of the bookshelf=864 square inches

the book shelf is a rectangular prism

if we have height=4b, width=3b, length=b

then the area=length * width

A=(l*w)*2 ( we have 2 shelves)

864=(b*3b)*2

864=6b²

b²=864/6=144

b=√144= 12 inches

4- to cover the sides :

(height * length)*2   ( we have 2 sides)

(4b*b)×2=1600

8b²=1600

b²=1600/8=200

b=√200=14.1

Answer:

Question #3:  b = 12 in

Question #4:  b = 14.1 in

Step-by-step explanation:

Please see in the image attached the actual proportions that the furniture manufacturer uses to build the furniture in question:

Height = 4 b

Width = 3 b

Depth = b

So for question #3, given that the customer wants a total surface of the shaded shelves to be 864 [tex]in^2[/tex]

we can write that one wants twice the area of each rectangle of width 3 b and depth b to total 864:

[tex]2\,(3b\,*\,b)=864\\6 b^2=864\\b^2=864/6\\b^2=144\\b=12\,\,in[/tex]

Question # 4:

The total lateral surface to be covered by the silk is 1600 [tex]in^2[/tex], therefore if we consider the surface of each lateral plank as:

Area of each lateral plank :

[tex](4b)\,(b) = 4\,b^2[/tex]

Then twice these is: [tex]8\, b^2[/tex]

So we can solve for be requesting that these total surface equal the amount of silk:

[tex]8\,b^2=1600\\b^2=1600/8\\b^2=200\\b=\sqrt{200} \\b\approx 14.1421\,\,in[/tex]

which rounded to the nearest tenth of an inch gives:

[tex]b\approx 14.1\,\,in[/tex]

You and your friend are playing a game. The two of you will continue to toss a coin until the sequence HH or TH shows up. If HH shows up first, you win. If TH shows up first, your friend wins. What is the probability of you winning?

Answers

Answer:

The probability of friend A winning with HH = 1/4.

Step-by-step explanation:

The probability of an event, A is P(A) given by the relationship;

P(A) = (The number of required outcome)/(The number of possible outcomes)

The parameters given are;

The condition of friend A winning = Coin toss sequence HH shows up

The condition of friend B winning = Coin toss sequence TH shows up

The number of possible outcomes = TT, TH, HH, HT = 4

(TH and HT are taken as different for the game to be fair)

The number of required outcome = HH = 1

Therefore;

The probability of friend A winning with HH = 1/4.

rewrite 1/5:1/2 as a unit rate

Answers

Hey there! I'm happy to help!

The unit rate is how much stuff there is per 1 unit. All ratios can be rewritten as fractions, this one could be 0.2/0.5. The word per means divide, and fractions are basically dividing. So, we want the denominator to be 1.

To get 0.5 to 1, we multiply by 2. So, we will multiply the fraction by 2/2.

0.2/0.5(2/2)=0.4/1

Therefore, the unit rate is 0.4/1 or 0.4:1 or 0.4 per 1.

Have a wonderful day! :D

find the exterior angle of a triangle whose interior opposite angles are 43 degree and 27 degree

Answers

Answer:

[tex]\huge\boxed{Exterior\ angle = 70\°}[/tex]

Step-by-step explanation:

The measure of exterior angle is equal to the sum of opposite interior angles.

So,

Exterior angle = 43+27

Exterior angle = 70°

Someone pls help thank you sm ..

Answers

Answer:

15 lawns

Step-by-step explanation:

The Xbox costs $400 and his parents gave him $100. He needs to earn $300 more. $20 each lawn so $100 for 5 lawns. 5 lawns times three.

Xbox cost =400

Daniel's money =100

Xbox cost - Daniel money=300

yard cost =20

therefore 400 -100 /20

Daniel's need 15 yards to get the xbox

Select the correct answer. This set of ordered pairs defines a function. {(-49,7), (-56,8), (-63,9), (-70,10)} Which table represents the inverse of the function defined by the ordered pairs? A.

Answers

In the future, you should post all possible answer choices to have a complete post. However, there's enough information to get the answer.

The original set has points in the form (x,y)

The first point is (x,y) = (-49,7) making x = -49 and y = 7. When we find the inverse, we simply swap the x and y values. The inverse undoes the original function and vice versa. So if (-49, 7) is in the original function, then (7, -49) is in the inverse. The rest of the points follow the same pattern.

We end up with this answer

{ (7, -49), (8, -56), (9, -63), (10, -70) }

please help i beg plsssssssssz​

Answers

Answer:

5/2=20/8=35/14=125/50

Answer:

8, 14, 50

Step-by-step explanation:

5 x 4 = 20

2 x 4 = 8

5 x 7 = 35

2 x 7 = 14

5 x 25 = 125

2 x 25 = 50

In the diagram of the right triangle shown find the value of c.​

Answers

Answer:

Hey there!

20^2+25^2=c^2

400+625=c^2

1025=c^2

Square root 1025 is the correct answer, so option C.

Let me know if this helps :)

Answer: B

Step-by-step explanation:

When the solution of x2 − 9x − 6 is expressed as 9 plus or minus the square root of r, all over 2, what is the value of r? X equals negative b plus or minus the square root of b squared minus 4 times a times c, all over 2 times a 6 42 57 105

Answers

Answer:

The value of r is 105

Step-by-step explanation:

Quadratic equation helps us to solve any quadratic equation. The formula is:

(-b ± √b² - 4ac) ÷ 2a

For the quadratic equation ax²+bx+c

For the equation: X² - 9x - 6:

a = 1, b = -9 and c = -6

Replacing in the quadratic formula:

(-(-9) ± √(-9)² - 4(1*-6)) ÷ 2*1

= 9 ± √105 / 2

That means, the value of r is 105

Answer:

105 is right

Step-by-step explanation:

I took the test

HELP ASAP WILL MARK BRAINLIEST!!!!!! Use the number line​ below, where ​RS=9y+2, ST = 4y+9 and RT = 115. a. What is the value of​ y? b. Find RS and ST. a. What is the value of​ y?

Answers

Answer: y = 7

Step-by-step explanation:

The circle shown below is a unit circle, where ∠a=π/3 and the radius of the circle is 1.

Answers

Answer:

Step-by-step explanation:

Pregunta N° 1: ¿Cuántas fracciones propias e irreductibles con denominador 24 existen? 1 punto A) 2 B) 4 C) 6 D) 8 E) 10 Pregunta N° 2: ¿Cuántas fracciones impropias e irreductibles con numerador 25 existen? 1 punto A) 19 B) 21 C) 25 D) 29 E) 33 Pregunta N° 3: La edad de Miguel es 4/5 de la edad de su novia. Si las edades de los dos suman 63 años, calcule la edad de la novia de Miguel. 1 punto A) 20 años B) 26 años C) 32 años D) 35 años E) 40 años Pregunta N° 4: Si son las 8 a. m., ¿qué fracción del día ha transcurrido? 1 punto A) 1 B) 2 C) 1/2 D) 1/3 E) 1/5

ayuden porfavor

Answers

Answer:

Pregunta 1: Opcion D. 8

Pregunta 2: Opción A. 19 (aunque lo correcto es decir que son 20)

Pregunta 3: 28 años (no está como opción)

Pregunta 4: Opción D. 1/3

Step-by-step explanation:

Las fracciones irreductibles son aquellas que después de dividirlas por un común divisor, una vez que no se pueden dividir más se dice que son irreducibles, por lo tanto no existe ningún número que sea divisor común del numerador y del denominador más que 1.

Fracciones irreductibles con común denominador 24.

Como máximo divisor tenemos el 24 y como mínimo el 1

entre 1/24 y 1 estarán nuestras fracciones o sea:

1/24 < x/24 < 1. Ahora convertimos el 1 en fracción de 24, lo que sería 24/24 para igualar el numerador en ambos lados de la ecuación, para poder determinar x

1/24 < x/24 < 24/24

Como vemos que x tiene que estar entre 1 y 24, las respuestas serán:

2, 3, 4, 5, 6, 7, 8, 9, 10, 11, 12, 13, 14, 15, 16, 17, 18, 19, 20, 21, 22 y 23

Eliminamos los números divisores de 24, aquellos pares, y nos focalizamos en los que no podriamos dividir por nada con 24, o sea los números primos

5, 7, 11, 13, 17, 19, 23. Como nos falta el 1, obtenemos un total de 8 fracciones: 1/24, 5/24, 7/24, 11/24, 13/24, 17/24, 19/24, 23/24

Mismo procedimiento para el 25:

1/25 es una de las fracciones irreductibles. Pensamos en los valores de x

1/25 < x/25 < 25/25

2, 3, 4, 5, 6, 7, 8, 9, 10, 11, 12, 13, 14, 15, 16, 17, 18, 19, 20, 21, 22, 23, 24, 25

Los números divisibles por 25, son los multiplos de 5, asi que esas respuestas no irían. Las fracciones irreductibles son:

1/25, 2/25, 3/25, 4/25, 6/25, 7/25, 8/25, 9/25, 11/25, 12/25, 13/25, 14/25, 16/25, 17/25, 18/25, 19/25, 21/25, 22/25, 23/25 y 24/25 haciendo un total de

20. Por alguna razón está mal formulada la pregunta, son 20 pero no está como opción y como te piden fraccion impropia (numerador > denominador), contamos a partir de 26. FIjate que hasta el proximo entero que sería 50/25, también son 20 fracciones (irreductibles e impropias)

26/25, 27/25, 28/25, 29/25, 31/25, 32/25, 33/25, 34/25, 36/25, 37/25, 38/25, 39/25, 41/25, 42/25, 43/25, 44/25, 46/25, 47/25, 48/25, 49/25

Próxima pregunta:

Miguel tiene 4/5 de la edad de la novia, y ambas edades suman 63.

Plantiemos la siguiente ecuacion donde x es la edad de la novia

4/5x + x = 63

9/5x = 63

x = 63 . 5/9 (como 9/5 pasa al otro lado de la igualdad dividiendo, damos vuelta la fraccion multiplicandola)

x = 35

Si la novia tiene 35 años y la edad de Miguel es 4/5 de esa edad

4/5 .35 = (35 .4) /5 = 28

Es raro porque no está la respuesta como tal.

Próxima pregunta:

Al ser las 8 am, quiere decir que han pasado 8 horas de que empezó el día

y el día tiene 24 horas.

8 horas transcurridas / 24 horas totales = 1/3

If Eric can paint $3$ cars in $4$ hours and $2$ trucks in $5$ hours, then how long, in hours, would it take him to paint $4$ cars and a truck? Express your answer as a common fraction.

Answers

Answer:

47/6

Step-by-step explanation:

Given that :

Time taken to paint 3 cars = 4 hours

Time taken to paint 2 trucks = 5 hours

How long will it take him to paint 4 cars and a truck

If 3 cars = 4 hours ;

Then ;

1 car = (4/3)hours

If 2 trucks = 5 hours

Then;

1 truck = (5/2) hours = 2 1/2 hours

Time required To paint 4 cars :

4 × (4/3) = 16/3 hours

Time required to paint 1 truck :5/2 hours

Total time required :

(16/3 + 5/2) = (32 + 15) / 6 = 47/6

To get from home to work, Felix can either take a bike path through the rectangular park or ride his bike along two sides of the park. How much farther would Felix travel by riding along two sides of the park than he would by taking the path through the park?

Answers

Answer:

c=5.9/6(G)

Step-by-step explanation:

first find the 2 distances.

a^2+b^2=c^2                    c=2.4+.7              

7^2+2.4^2=c^2                  c=3.1

.49+5.85=c^2                    

c^2=6.34

c=√6.34

c=2.51.

next subtract the two distances to find the difference.

c=2.51-3.1

c=.59

so the distance would be .59 which can be rounded up to .60/G

explanation on how I knew the answer.

Im reviewing for the math 8th grade staar.

nine hundred fifty-three thousand nine hundred two

Answers

Answer:

953 902

Step-by-step explanation:

Answer:

953,902

Step-by-step explanation:

nine hundred =900

fifty-three=53

thousand=1000

nine hundred=900

two=2

a broker gets rs 20000 as commission from sale of a piece of land which costs rs 8000000. Find the rate of commission.​

Answers

Answer:

0.25%

Step-by-step explanation:

Rate of commission

= (commission*100)/cost of land

=( 20000*100)/8000000

= 2000000/8000000

=2/8

= 0.25%


someone please expain how to do this, i’m really confused.

Answers

Answer:

13

Step-by-step explanation:

Basically, we have to plug in 4 for r into g(r). Doing so gives us g(4) = 25 - 3 * 4 = 25 - 12 = 13.

Some more examples:

g(6) = 25 - 3 * 6 = 25 - 18 = 7

g(1) = 25 - 3 * 1 = 25 - 3 = 22

Answer:g(4)=13

Step-by-step explanation:

g(4)=25-3r

25-3(4)

25-12

g(4)=13

A line passes (-8,-2) and has a slope of 5/4. Write an equation in Ax + By=C

Answers

Answer:

5x-4y = -32

Step-by-step explanation:

First write the equation in point slope form

y-y1 = m(x-x1)

y - -2 = 5/4 ( x- -8)

y+2 = 5/4 (x+8)

Multiply each side by 4 to clear the fraction

4( y+2 )= 4*5/4 (x+8)

4y +8 = 5(x+8)

4y+8 = 5x+40

Subtract 4y from each side

8 = 5x-4y +40

Subtract 40 from each side

-32 = 5x-4y

5x-4y = -32

Answer:

The answer is

5x - 4y = -32

Step-by-step explanation:

To write an equation of a line given a point and slope use the formula

y - y1 = m( x - x1)

where

m is the slope

( x1 , y1) is the point

From the question

slope = 5/4

point (-8 , -2)

So the equation of the line is

[tex]y + 2 = \frac{5}{4} (x + 8)[/tex]

Multiply through by 4

4y + 8 = 5( x + 8)

4y + 8 = 5x + 40

5x - 4y = 8 - 40

We have the final answer as

5x - 4y = -32

Hope this helps you

WILL GIVE BRAINLIEST!!!!!! Look at the picture of a scaffold used to support construction workers. The height of the scaffold can be changed by adjusting two slanting rods, one of which, labeled PR, is shown: Part A: What is the approximate length of rod PR? Round your answer to the nearest hundredth. Explain how you found your answer, stating the theorem you used. Show all your work. Part B: The length of rod PR is adjusted to 17 feet. If width PQ remains the same, what is the approximate new height QR of the scaffold? Round your answer to the nearest hundredth. Show all your work.

Answers

A) Here, We'll use "Pythagoras Theorem" which tells:

a² + b² = c²

So, PR² = PQ² + QR²

PR² = 14² + 9²

PR² = 196 + 81

PR = √277

In short, Your Answer would be 16.64 Feet

B) Again, Use the Pythagoras Theorem,  

c² - a² = b²

18² - 14² = b²

b² = 324 - 196

b = √128

b = 11.31

In short, Your Answer would be 11.31 Feet

Part A: Using the Pythagorean Theorem. a^2 + b^2 = c^2

PQ^2 + QR^2 = PR^2  (The rods make a right triangle, where PR would be the hypotenuse, and QR and PQ would be legs a and b.)

14^2 + 9^2 = PR^2

196 + 81 = PR^2

Square root of 277 = PR 

16.64 = PR

So, the hypotenuse would be equal to 16.64 ft.

Part B: Using the Pythagorean Theorem. a^2 + b^2 = c^2

PR^2 - PQ^2 = QR^2 (Trying to find the height of QR this time, not the hypotenuse, since we know what it is already. Subtracting the value of leg a from the hypotenuse will give us the value of leg b, QR.)

18^2 - 14^2 = QR^2

324 - 196 = QR^2

Square root of 128 = QR

So, the new height of QR would be 11.31 ft.


Hey, please help solve the question.​

Answers

Answer:

75%=x-125

90%=x+250

subtract the second from the first

15%=375

100%=?

100%×375/15

100%=2500marked price is 2500

2500+250=2750

90%=2750

100%=?

cost price=3055.56

Sarah has $20 saved. She gets $10 per week for her allowance, and she saves her allowance for the next 3 weeks. At the end of the week, she gets $150 in birthday money. How much money will she have after the 3 weeks? Which of the following sets of equations represents this problem?

Answers

Answer:

$200

Step-by-step explanation:

We know that she already has $20. And we know that every week, for three weeks she gets $10.

20+3(10)+150=m

We add all of this up, and we find that at the end of 3 weeks Sarah has $200 saved.

Why the answer question now correct

Answers

Answer:

461.58 in²

Step-by-step explanation:

The surface area (A) is calculated as

A = area of base + area of curved surface

   = πr² + πrl ( r is the radius of base and l is slant height )

    = 3.14 × 7² + 3.14 × 7 × 14

    = 3.14 × 49 + 3.14 × 98

    = 3.14(49 + 98)

    = 3.14 ×147

     = 461.58 in²

solve the following: - 3 raised to 1 by 5 the whole raised to 4 (3^1/5)^4

Answers

Answer:

8.30256

Step-by-step explanation:

Step 1: Write out expression

[tex]((-3)^{\frac{1}{5} })^{4(3^{\frac{1}{5} })^4[/tex]

Step 2: Use BPEMDAS to evaluate

[tex](-1.24573)^{4(3^{\frac{1}{5} })^4[/tex]

[tex](-1.24573)^{4(1.24573)^4[/tex]

[tex](-1.24573)^{4(2.40822)[/tex]

[tex](-1.24573)^{9.6329}[/tex]

= 8.30256

And we have our answer!

Other Questions
is this a function {(-2, 6), (-3, 7), (-4, 8), (-3, 10)} Nous finions nos devoirs Find the vertex of f(x)= x^2+ 6x + 36Pls help soon According to the passage, what is one purpose of dance rituals in Iroquois culture? Simon Corporation manufactures hydraulic valves. The product life of a valve is 4 years. Target average profit margin for Simon 20.00% The company does not expect the manufacturing cost to vary over the next 4 years. Estimated sales volume and the unit selling price of the valve for the next 4 years is given below: Year Sales volume (units) Unit selling price Year 1 40,000 $80.00 Year 2 50,000 $75.00 Year 3 35,000 $50.00 Year 4 25,000 $45.00 What is the allowable unit cost of a hydraulic valve using the target costing model what effect does decreasing the field current below its nominal value have on the speed versus voltage characteristic of a separately excited dc motor Given the equations of a straight line f(x) (in slope-intercept form) and a parabola g(x) (in standard form), describe how to determine the number of intersection points, without finding the coordinates of such points. Do not give an example. Jamie was participating in a market research study regarding computers when he was presented with 24 different computers that varied on four criteria. He was asked to rank all 24 descriptions in terms of his preference for those combinations of features. Which approach to assess the relative importance Jamie places on evaluative criteria was this research using? Shaquira is baking cookies to put in packages for a fundraiser. Shaquira has made 86 8686 chocolate chip cookies and 42 4242 sugar cookies. Shaquira wants to create identical packages of cookies to sell, and she must use all of the cookies. What is the greatest number of identical packages that Shaquira can make? good is excludable if: a. it is Wi-Fi or a similar service. b. people who do not pay cannot be easily prevented from using the good. c. one person's use of the good does not reduce the ability of another person to use the same good. d. people who do not pay can be easily prevented from using the good. Find all values of x on the graph of f(x) = 2x3 + 6x2 + 7 at which there is a horizontal tangent line. The value of y varies jointly with x and z. If y = 7 when z = 196 and x = 2, find the value of y when x = 3 and z = 336. I will rate you brainliest Find the value of mACD. A. 30 B. 15 C. 60 D. 90 1. While FF was started 40 years ago, its common stock has been publicly traded for the past 25 years. 2. The returns on its equity are calculated as arithmetic returns. 3. The historical returns for FF for 2012 to 2016 are: Most energy enters ecosystems in the form of sunlight,Please select the best answer from the choices providedTF me pleasee, thank you Compute the flux of the vector field LaTeX: \vec{F}=F =< y + z , x + z , x + y > though the unit cubed centered at origin. 4. Why does every decision involve trade-offs? A sports club was formed in the month of May last year. The function below, M(t), models the number of club members for the first 10 months, where t represents the number of months since the club was formed in May. m(t)=t^2-6t+28 What was the minimum number of members during the first 10 months the club was open? A. 19 B. 28 C. 25 D. 30 Policeman A and Policeman B hand out 70 speeding tickets in a month.Policeman A hands out 4 times as many speeding tickets as Policeman B.Policeman A handed out ? Speeding tickets.